Using parametric differentiation to evaluate the slope of a curve - attempted

Click For Summary
The discussion revolves around evaluating the slope of a curve defined by parametric equations at the point t=1. The user correctly finds the coordinates (0, 1) by substituting t=1 into the equations for x(t) and y(t). They calculate dy/dt and dx/dt using the Quotient Rule but encounter a zero numerator when substituting t=1, leading to confusion about the slope. It is clarified that at t=1, the tangent line is horizontal, resulting in a slope of zero, which is consistent with the graph of the function being an ellipse. The user is encouraged to plot the graph to visualize the behavior of the curve.
chuffy
Messages
22
Reaction score
0

Homework Statement



x(t) = (t^2 -1) / (t^2 +1)

y(t) = (2t) / (t^2 +1)

at the point t=1

Homework Equations



Line equation = y-y1 = m(x-x1)

chan rule = (dy/dt) / (dx/dt) = dy/dx

The Attempt at a Solution



I find the y1 and x1 values by subing in t=1 to the x(t) and y(t) equations
I get the point (0,1) when t=1

I have used the Quotient rule to find dx/dt & dy/dx (Is this right?)
Doing the above I get:

dy/dt = (-2(t^2 -1)) / (t^2+1)^2

dx/dt = (4t) / (t^2 +1)^2

So dy/dx = (dy/dt) / (dx/dt) however when I sub in t=1 to (dy/dt) I get 0 as the numerator

I know that the m of the line equation is equal to dy/dx

does anyone know what I'm doing wrong? I'll try uploading a pic of my work
cheers
 
Physics news on Phys.org
pic of my working
 

Attachments

  • IMG_20130207_190038.jpg
    IMG_20130207_190038.jpg
    35.3 KB · Views: 410
What makes you think something is wrong? have you plotted the x v y graph for t having a range of values? say t=-1, 0, 1, 2, 3
 
if t=1 does this mean that m= 0?
 


The graph of this function is an ellipse. t= 1 is the point (0, 1) where the tangent line is, in fact, horizontal so the dy/dx= 0 there.
 

Attachments

  • ellipse.jpeg
    ellipse.jpeg
    8.2 KB · Views: 468


cheers
 
Question: A clock's minute hand has length 4 and its hour hand has length 3. What is the distance between the tips at the moment when it is increasing most rapidly?(Putnam Exam Question) Answer: Making assumption that both the hands moves at constant angular velocities, the answer is ## \sqrt{7} .## But don't you think this assumption is somewhat doubtful and wrong?

Similar threads

  • · Replies 8 ·
Replies
8
Views
1K
  • · Replies 2 ·
Replies
2
Views
1K
Replies
3
Views
2K
  • · Replies 20 ·
Replies
20
Views
2K
  • · Replies 9 ·
Replies
9
Views
2K
  • · Replies 3 ·
Replies
3
Views
2K
Replies
12
Views
2K
  • · Replies 2 ·
Replies
2
Views
2K
  • · Replies 6 ·
Replies
6
Views
2K
Replies
2
Views
1K